All Exams  >   GRE  >   GRE Mock Test Series 2025  >   All Questions

All questions of Practice Tests for GRE Exam

Directions: After reading the passage, read and respond to the questions that follow by selecting the best choice for each one.
[1] An atmospheric gas that absorbs and emits thermal radiation in the infrared range is known as a greenhouse gas. [2] Without such gases, the surface temperature of the earth would have been a frigid -18°C instead of the current 15°C. [3] Many climate scientists claim that human activities since the beginning of the Industrial Revolution have led to a steady increase in the atmospheric concentration of many greenhouse gases, with a consequent increase in the planet's surface temperature by about 0.85°C in the last 130 years. [4] However, in order to correctly estimate the anthropogenic changes in earth's surface temperature, it is important to quantify the effect of external natural factors like large volcanic eruptions on the planet's surface temperature.
[5] Though volcanic eruptions emit carbon dioxide, a greenhouse gas, the atmospheric concentration of this gas is about 16000 times the amount released by volcanic eruptions; therefore, no matter how large a volcanic eruption is, it cannot produce a significant change in the atmospheric carbon dioxide levels. [6] Further, the total amount of carbon dioxide emitted per year by volcanic eruptions – between 130 and 230 million tons – is roughly equivalent to that emitted in only three to five days of human activity.
[7] It is mainly through the emission of sulfur dioxide gas and ash particles into the atmosphere that large volcanic eruptions affect earth's surface temperature. [8] Sulfur dioxide reacts with the water vapor present in air to form fine particles called sulfate aerosols. [9] Winds spread the cloud of aerosols and ash particles around the globe in weeks. [10] These particles absorb incoming solar radiation and scatter it back into space, thereby producing a cooling effect on the earth. [11] They take several years to settle out of the atmosphere and thus impact the global surface temperature for many years. [12] Robock and Mao have shown that for two years after a great volcanic eruption, the surface temperature decreases by 0.1 – 0.2°C. [13] The 1991 Pinatubo eruption in Indonesia was one of the largest volcanic eruptions in the twentieth century and resulted in a global surface cooling of 0.5°C for about two to four years after the eruption.
[14] The El Chichón eruption in 1982 was the first major eruption whose climatic impact was studied in detail by modern instruments. [15] Though the emission volume of this eruption was similar to the Mount St. Helens eruption in 1980, El Chichón released seven times the amount of sulfate aerosols released by Mount St. Helens and lowered the earth's temperature by about 0.4°C as against a decrease of only 0.1°C for Mount St. Helens.
Q. The facts presented in the last paragraph of the passage support which of the following conclusions?
  • a)
    The carbon content in the volcanic ash particles determines the extent of global surface cooling done by the eruption.
  • b)
    The decrease in earth's surface temperature upon a volcanic eruption is determined by the percentage of sulfur dioxide in the total volcanic emissions.
  • c)
    Sulfate aerosols take longer to settle out of the atmosphere than ash particles.
  • d)
    The sulfate aerosols produced during an eruption have a greater effect on earth's surface temperature than the ash produced during the eruption.
  • e)
    The longevity of a volcanic eruption's impact on earth's surface temperature depends on the amount of sulfur dioxide emitted by it.
Correct answer is option 'D'. Can you explain this answer?

Let us revise the table drawn for the last paragraph in the passage analysis section
From this table, we may conclude that the greater the amount of sulfate aerosols emitted by a volcano, the greater the decrease in surface temperature.
Let us analyze the options one by one.
  1. This option is incorrect. It is not the carbon content in the ash but the amount of sulfate aerosols that determines the extent of global surface cooling done by the eruption.
  2. This option is incorrect. Can we say that a small volcanic eruption in which sulfur dioxide is 80 percent of the emitted matter will produce more cooling than a large volcanic eruption in which sulfur dioxide is 20 percent of the emitted matter? No, the passage gives us no basis to make such a claim. Further, the passage does not even tell us the percentage of sulfur dioxide in the El Chichón and Mount St. Helens emissions (that is, the value of (7s / t) and (s / t) respectively).
  3. This option is incorrect. The last paragraph does not compare, or even mention, the settling times of sulfate aerosols and ash particles.
  4. This option is correct. This is in line with the analysis done above.
  5. This option is incorrect. The last paragraph does not mention the factors that impact how long a volcanic eruption's impact on earth's surface temperature remains.

Direction: Select exactly two answer choices that best complete the sentence and produce sentences that are alike in meaning.
Q. Water experts predict that unless the coming year’s rainfall will be significantly above average, the city’s denizens, regardless of any protestations, will have to _____ their water usage.
  • a)
    curtail
  • b)
    intensify
  • c)
    administer
  • d)
    denote
  • e)
    limit
Correct answer is option 'A,E'. Can you explain this answer?

Meera Rana answered
The sentence makes clear that rain is not likely. The denizens, or inhabitants, therefore have to reduce or limit their water usage. As curtail means “to limit,” both (A) curtail and (E) limit are correct.
(B) intensify is the opposite
(C) administer does not make sense
(D) denote means to make or indicated

Directions: Select all the answer choices that apply.
Q. The penguins currently living on an island are of two types, Chinstrap penguins and Gentoo penguins. The range of the heights of the Chinstrap penguins on the island is 13.2 centimeters, and the range of the heights of the Gentoo penguins on the island is 15.4 centimeters.
Which of the following statements individually provide(s) sufficient additional information to determine the range of the heights of all the penguins on the island?
Indicate all such statements.
  • a)
    The tallest Gentoo penguin on the island is 5.8 centimeters taller than the tallest Chinstrap penguin on the island.
  • b)
    The median height of the Gentoo penguins on the island is 1.1 centimeters greater than the median height of the Chinstrap penguins on the island.
  • c)
    The average (arithmetic mean) height of the Gentoo penguins on the island is 4.6 centimeters greater than the average height of the Chinstrap penguins on the island.
Correct answer is option 'A'. Can you explain this answer?

Choice A tells you that the tallest Gentoo penguin is 5.8 centimeters taller than the tallest Chinstrap penguin. You can combine this information with the given information about the Gentoo and Chinstrap height ranges to place four penguins—the shortest Gentoo, the shortest Chinstrap, the tallest Gentoo, and the tallest Chinstrap— in relative order according to height, as shown in the figure below.
You can see from the figure that the tallest penguin must be a Gentoo and the shortest penguin must be a Chinstrap. You can also see the difference in height between those two penguins, which is the range of the heights of the all the penguins. Therefore, Choice A provides sufficient additional information to determine the range.
Choice B provides information about one of the centers of the data—the median; it does not say anything about how spread out the data are around that center. You are given that the median height of the Gentoos is 1.1 centimeters greater than that of the Chinstraps. First note that it is possible for two different sets of data to have the same median but have very different ranges. Choice B gives the difference between the medians of the Gentoo heights and the Chinstrap heights, without giving the actual medians. However, even if you knew the medians, the fact that the ranges can vary widely indicates that the range of the heights of the all the penguins can also vary widely.
It is possible to construct examples of heights of penguins that satisfy all of the information in the question and in Choice B but have different ranges for the heights of the all the penguins. Here are two such examples, each of which has only three Chinstraps and three Gentoos. Although the examples are small, they illustrate the fact that the range of the heights of the all the penguins can vary. In both examples, the range of Chinstrap heights is 13.2 centimeters, the range of Gentoo heights is 15.4 centimeters, and the difference between the median heights is 1.1 centimeters.
Therefore, Choice B does not provide sufficient additional information to determine the range of the heights of the all the penguins.
Choice C provides information about another center of the data—the average. You are given that the average height of the Gentoos is 4.6 centimeters greater than that of the Chinstraps. However, like Choice B, the statement gives no information about how spread out the data are around that center. Again, it is possible for two different sets of data to have the same average but have very different ranges. Examples similar to the two examples above can be constructed that satisfy all of the information in the question and in Choice C but have different ranges for the heights of the all the penguins. Therefore, Choice C does not provide sufficient additional information to determine the range of the heights of the all the penguins.
The correct answer consists of Choice A.

Direction: Read the following passage carefully and answer the questions given below it. 
Parents of high-school students argue that poor attendance is the result of poor motivation. If students’ attitudes improve, regular attendance will result. The administration, they believe, should concentrate less on making stricter attendance policies and more on increasing students’ learning.
Q. Which of the following, if true, would most effectively weaken the parents’ argument?
  • a)
    Motivation to learn can be improved at home, during time spent with parents.
  • b)
    The degree of interest in learning that a student develops is a direct result of the amount of time he or she spends in the classroom.
  • c)
    Making attendance policies stricter will merely increase students’ motivation to attend classes, not their interest in learning.
  • d)
    Showing a student how to be motivated is insufficient; the student must also accept responsibility for his or her decisions.
  • e)
    Unmotivated students do not perform as well in school as other students.
Correct answer is option 'B'. Can you explain this answer?

Nandini Rao answered
Explanation:

Amount of time spent in the classroom:
- If the degree of interest in learning is directly related to the amount of time spent in the classroom, then poor attendance could indeed be a result of poor motivation.
- This weakens the parents' argument because it suggests that stricter attendance policies might actually be necessary to increase students' interest in learning.
Therefore, option B is the most effective in weakening the parents' argument as it implies that poor attendance could be a direct result of poor motivation, rather than the other way around.

Directions: After reading the passage, read and respond to the questions that follow by selecting the best choice for each one.
[1] An atmospheric gas that absorbs and emits thermal radiation in the infrared range is known as a greenhouse gas. [2] Without such gases, the surface temperature of the earth would have been a frigid -18°C instead of the current 15°C. [3] Many climate scientists claim that human activities since the beginning of the Industrial Revolution have led to a steady increase in the atmospheric concentration of many greenhouse gases, with a consequent increase in the planet's surface temperature by about 0.85°C in the last 130 years. [4] However, in order to correctly estimate the anthropogenic changes in earth's surface temperature, it is important to quantify the effect of external natural factors like large volcanic eruptions on the planet's surface temperature.
[5] Though volcanic eruptions emit carbon dioxide, a greenhouse gas, the atmospheric concentration of this gas is about 16000 times the amount released by volcanic eruptions; therefore, no matter how large a volcanic eruption is, it cannot produce a significant change in the atmospheric carbon dioxide levels. [6] Further, the total amount of carbon dioxide emitted per year by volcanic eruptions – between 130 and 230 million tons – is roughly equivalent to that emitted in only three to five days of human activity.
[7] It is mainly through the emission of sulfur dioxide gas and ash particles into the atmosphere that large volcanic eruptions affect earth's surface temperature. [8] Sulfur dioxide reacts with the water vapor present in air to form fine particles called sulfate aerosols. [9] Winds spread the cloud of aerosols and ash particles around the globe in weeks. [10] These particles absorb incoming solar radiation and scatter it back into space, thereby producing a cooling effect on the earth. [11] They take several years to settle out of the atmosphere and thus impact the global surface temperature for many years. [12] Robock and Mao have shown that for two years after a great volcanic eruption, the surface temperature decreases by 0.1 – 0.2°C. [13] The 1991 Pinatubo eruption in Indonesia was one of the largest volcanic eruptions in the twentieth century and resulted in a global surface cooling of 0.5°C for about two to four years after the eruption.
[14] The El Chichón eruption in 1982 was the first major eruption whose climatic impact was studied in detail by modern instruments. [15] Though the emission volume of this eruption was similar to the Mount St. Helens eruption in 1980, El Chichón released seven times the amount of sulfate aerosols released by Mount St. Helens and lowered the earth's temperature by about 0.4°C as against a decrease of only 0.1°C for Mount St. Helens.
Q. Select a sentence in the passage that, if true, supports the claim that the primary cause of the steady increase in the atmospheric concentration of greenhouse gases is not volcanic eruptions but human activities.
  • a)
    Sentence 3
  • b)
    Sentence 4
  • c)
    Sentence 6
Correct answer is option 'C'. Can you explain this answer?

Meera Rana answered
Human activities are mentioned thrice in the passage:
In the first paragraph, they are mentioned first in the claim made ("Many climate ... last 130 years.") by the climate scientists, and then in the author's suggestion that in order to correctly estimate the impact of human activities on surface temperature increase ("However, in … surface temperature."), we also need to quantify the impact of external natural factors like large volcanic eruptions. Neither of these two mentions suit the purpose of this question.
The final mention of human activities is in the last sentence of the second paragraph ("Further, the total amount . . . human activity"). As also discussed in the passage analysis above, in this sentence, the author suggests that the carbon dioxide emissions from volcanic eruptions are only of the annual carbon dioxide emissions from human activity. Therefore, the author is suggesting that humans make a far more significant increase in atmospheric carbon dioxide concentration, and therefore in the resulting surface temperature increase, than volcanic eruptions.

Directions: Compare Quantity A and Quantity B, using additional information centered bove the two quantities if such information is given. Select one of the following four answer choices. A symbol that appears more than once in a question has the same meaning throughout the question.
Q. Machine R, working alone at a constant rate, produces x units of a product in 30 minutes, and machine S, working alone at a constant rate, produces x units of the product in 48 minutes, where x is a positive integer.
Quantity A: The number of units of the product that machine R, working alone at its constant rate, produces in 3 hours
Quantity B: The number of units of the product that machine S, working alone at its constant rate, produces in 4 hours
  • a)
    Quantity A is greater.
  • b)
    Quantity B is greater.
  • c)
    The two quantities are equal.
  • d)
    The relationship cannot be determined from the information given.
Correct answer is option 'A'. Can you explain this answer?

Stella Lopez answered
Quantity A: The number of units of the product that machine R, working alone at its constant rate, produces in 3 hours.

Quantity B: The number of units of the product that machine S, working alone at its constant rate, produces in 4 hours.

To compare Quantity A and Quantity B, we need to calculate the number of units produced by each machine in the given time periods.

Calculating Quantity A:
Machine R produces x units of the product in 30 minutes. Therefore, in 1 hour (60 minutes), it would produce 2x units (since 60/30 = 2). In 3 hours, it would produce 6x units (since 3 * 2x = 6x).
So, Quantity A is 6x units.

Calculating Quantity B:
Machine S produces x units of the product in 48 minutes. Therefore, in 1 hour (60 minutes), it would produce (60/48) * x units. Simplifying this, we get (5/4) * x units. In 4 hours, it would produce 4 * (5/4) * x units. Simplifying this, we get 5x units.
So, Quantity B is 5x units.

Comparing Quantity A and Quantity B:
Quantity A = 6x units
Quantity B = 5x units

Since 6x is greater than 5x for any positive value of x, we can conclude that Quantity A is greater than Quantity B. Therefore, the correct answer is option 'A' - Quantity A is greater.

Summary:
- Quantity A represents the number of units produced by machine R in 3 hours, and Quantity B represents the number of units produced by machine S in 4 hours.
- By calculating the number of units produced in the given time periods, we find that Quantity A is 6x units and Quantity B is 5x units.
- Since 6x is greater than 5x for any positive value of x, we can conclude that Quantity A is greater than Quantity B.

Directions: Answer the questions based on following reading passage.
Arctic sea ice comes in two varieties. Seasonal ice forms in winter and then melts in summer, while perennial ice persists year-round. To the untrained eye, all sea ice looks similar, but by licking it, one can estimate how long a particular piece has been floating around. When ice begins to form in seawater, it forces out salt, which has no place in the crystal structure. As the ice gets thicker, the rejected salt collects in tiny pockets of brine too highly concentrated to freeze. A piece of first-year ice will taste salty. Eventually, if the ice survives, these pockets of brine drain out through fine, veinlike channels, and the ice becomes fresher; multiyear ice can even be melted and drunk.
Q. The passage mentions which of the following as being a characteristic of seasonal ice?
[For the following question, consider each of the choices separately and select all that
apply.]
  • a)
    It is similar in appearance to perennial ice.
  • b)
    It is typically filled with fine, veinlike channels.
  • c)
    It tastes saltier than perennial ice.
Correct answer is option 'A,C'. Can you explain this answer?

Meera Rana answered
Choices A and C are correct.
Choice A is correct: the passage states that “to the untrained eye, all sea ice looks similar” (lines 2–3).
Choice B is incorrect: it is clear that perennial ice contains fine, veinlike channels, but the passage does not mention whether seasonal ice contains them.
Choice C is correct: in lines 6–8, the passage establishes that first-year ice tastes salty but eventually gets fresher if the ice survives.

Direction: Read the following passage carefully and answer the questions given below it.
Pancreatic beta cells are responsible within a body for monitoring homeostatic cues from a wide variety of hormonal inputs and in turn regulate the insulin needed to maintain balance in the blood sugar. Researchers studying this signaling system have located three key proteins that relay signals. Understanding how these proteins function within the context of the signaling system can help scientists gain more insight into how diabetes compromises the healthy functioning of the system and how to counteract the impact of diabetes once it is identified.
Q. The passage implies which of the following about diabetes research?
  • a)
    When scientists fully understand how these key proteins work, rates of Type II diabetes will be greatly reduced.
  • b)
    Researchers hope to understand the signaling system of pancreatic beta cells in order to replicate an artificial system.
  • c)
    Without a complete understanding of these key proteins, diabetes research is at a standstill.
  • d)
    Future medications for diabetes may include or simulate some of the hormonal cues that pancreatic beta cells monitor.
  • e)
    One way researchers hope to fight diabetes is by stopping the relay signal system in the pancreas by cutting off the proteins.
Correct answer is option 'D'. Can you explain this answer?

Meera Rana answered
The correct answer follows from the passage, though it may not be stated explicitly. Choice (A) is not implied. Although researchers probably hope to stem the occurrence of diabetes, the passage speaks only of combating the disease’s impact. There is also no suggestion in the passage of the hope, expressed in choice (B), of creating an artificial system. Choice (C) is too extreme. The passage doesn’t imply a “standstill” absent “complete” understanding of the proteins. The wording in choice (D) is dense, but it’s also broad enough to follow from the passage. Researchers may well use their understanding of the hormonal cues monitored by the proteins to develop the treatments mentioned in the final sentence of the passage. Indeed, (D) is the correct response. Finally, (E) runs counter to the passage. The researchers hope to understand how to help the relay system work, not how to cut it off.
You can eliminate (E) confidently.

According to the table given, which brands contributed to more than 1/5 of the total sales in the first year? Indicate all applicable answers.
  • a)
    brands A, B and C
  • b)
    brands B, C and D
  • c)
    brands A, C and D
  • d)
    brands A and D
  • e)
    brands B and C
Correct answer is option 'C'. Can you explain this answer?

Meera Rana answered
First, compute the total sales for the first year
Total sales = 8.02 + 4.38 + 8.11 + 6.23 = 26.74
Even with sales expressed as 11,000,000 of their actual values, the results will remain consistent as long as all figures are expressed this way.
The question asks for the brand that had sales of more than 15 or 20% of the total sales for the year.
We may now compute for the % sales of each brand with respect to the total sales:
Clearly, car brands A, C and D each sold more than 20% or 1/5 of the total sales for the year.

In the figure below, O and P are the centers of the two circles. If each circle has radius r, what is the area of the shaded region?

 
  • a)
  • b)
  • c)
  • d)
  • e)
Correct answer is option 'B'. Can you explain this answer?

If a geometric problem contains a figure, it can be helpful to draw additional lines and add information given in the text of the problem to the figure. For circles, the helpful additional lines are often radii or diameters. In this case, drawing radius OP will divide the shaded region into two triangles, as shown in the figure below.
The two circles have the same radius, r. Therefore, in each of the triangles, all three sides have length r, and each of the triangles is equilateral. If you remember from geometry that the height of an equilateral triangle with sides of length r is √3/2r, you could use that fact in solving the problem. However, if you do not remember what the height is, you can use the following figure to help you find the height.
Using the Pythagorean theorem, you get
So the area of the equilateral triangle is 1/2(base)(height) =
Since the shaded region consists of 2 equilateral triangles with sides of length r, the area of the shaded region is , and the correct answer is Choice B.

In the square shown below, the side is 4 units. The three semicircles with their diameters along the sides of the square touch each other as shown. What is the diameter of the smaller semicircle?
  • a)
    √2 − 1
  • b)
    2√2 − 2
  • c)
    √2
  • d)
    4√2 − 4
  • e)
    √3
Correct answer is option 'D'. Can you explain this answer?


In the above diagram, S and R are the centers of the semicircles.
PS = SQ = QR = Half the side of the square = 2
Thus: ST = Radius of the bigger semicircle = 2
Let the radius of the smaller semicircle = r
⇒ SR = 2 + r
Thus, from Pythagoras' theorem in triangle SQR:
SR2 = SQ2 + QR2
⇒ (2 + r)2 = 22 + 22 = 8
Taking square-root throughout:
⇒ 2 + r = √8 = 2√2
⇒ r = 2√2 − 2
⇒ Diameter = 4√2 − 4
The correct answer is option D.

If (a−3)2 + ∣∣b − 3∣∣ = 0, what is the value of a − b?
    Correct answer is '0'. Can you explain this answer?

    We know that (a−3)2 + ∣∣b − 3∣∣ = 0
    We observe that (a−3)2 is a 'perfect square' term
    ⇒ (a − 3)2 ≥ 0
    Again, |b − 3| is an 'absolute value' term
    ⇒ |b − 3| ≥ 0
    Thus, we observe that two 'non-negative' terms add up to '0,' which is possible only if each term is '0.'
    Thus, we have (a − 3)= 0
    ⇒ a − 3 = 0
    ⇒ a = 3 … (i)
    Also, we have:
    |b − 3| = 0
    ⇒ b − 3 = 0
    ⇒ b = 3 … (ii)
    Thus, from (i) and (ii), we have a − b = 3 − 3 = 0

     what is the value of x2?
    • a)
      1
    • b)
      4
    • c)
      9
    • d)
      16
    • e)
      25
    Correct answer is option 'C'. Can you explain this answer?

    Meera Rana answered
    Given:

    We have to find out the value of x2.
    Let's first solve this with the traditional method and then we would solve it through alternate approaches.
    We have 
    Squaring both the sides, we get:

    Again, squaring both the sides, we get:
    4(x + 6)(x + 1) = (18 − 2x)2 
    4x+ 28x + 24 = 324 − 72x + 4x2
    100x = 300
    x = 3 ⇒ x2 = 9

    Directions: Select the two answer choices that, when inserted into the sentence, fit the meaning of the sentence as a whole and yield complete sentences that are similar in meaning.
    Q. Located in what is perhaps the most geographically remote state in the Union, Hawaii’s isolated Molokai Island is known for its hidden, untouched, and_____ beaches.
    • a)
      morose
    • b)
      immaculate
    • c)
      dilapidated
    • d)
      placid
    • e)
      pristine
    Correct answer is option 'B,E'. Can you explain this answer?

    You can infer from the words “isolated,” “hidden,” and “untouched” that the blank must be in line with these adjectives. It wouldn’t make sense for a beach to be hidden, untouched, and (A) morose, “gloomy and sullen,” or (C) dilapidated, “in disrepair or run-down,” so you can eliminate these choices. Choices (D) placid, “calm,” and But the sentence doesn’t rule out the possibility that the beaches could be less than placid, and just because a beach hasn’t been touched yet doesn’t mean it is immune from being disturbed in the future. You are le with choice (B) immaculate, “without stain or flaw,” and (E) pristine, “untouched” or “uncorrupted,” which are both synonyms of the adjectives used elsewhere in the sentence and therefore the correct answers.

    Direction: Read the following passage carefully and answer the questions given below it. 
    A property management company that operates a large townhouse complex has recently implemented a strict new policy that prohibits tenants from having certain breeds of dogs that it deems overly aggressive. The policy will apply only to new tenants and tenants who renew their leases. There has been a steep increase in the number of dog attacks associated with these breeds over the last two years, and the property manager has concluded that the new policy is the only way to reduce the number of attacks. The tenants’ association disagrees, claiming that the new policy will not significantly remedy the situation in the long run.
    Q. Which of the following, if true, would most effectively undermine the claim made by the tenants’ association?
    • a)
      Similar apartment complexes in the area have been able to effectively reduce the number of dog attacks.
    • b)
      There have been some dog attacks from breeds not added to the restricted list.
    • c)
      The apartment owner’s insurance policy does not cover any costs associated with dog attacks.
    • d)
      More than 90 percent of the current tenants have leases that expire over the next two years.
    • e)
      A majority of attacks occurred when tenants did not have proper control over their dogs.
    Correct answer is option 'D'. Can you explain this answer?

    Maya Carter answered
    Explanation:

    Current Tenants' Lease Expiry:
    - Option D states that more than 90 percent of the current tenants have leases that expire over the next two years.
    - This information is crucial as it shows that the new policy will affect a significant portion of the tenants in the complex.
    - If a large majority of tenants are affected by the policy, it can have a substantial impact on the overall situation.

    Undermining the Tenants' Association Claim:
    - If a majority of current tenants are subject to the new policy, it implies that the policy can have a significant long-term effect.
    - The tenants' association's claim that the new policy will not significantly remedy the situation in the long run is weakened by the fact that a large number of current tenants will be impacted.
    - This information suggests that the new policy could indeed have a lasting positive effect on reducing dog attacks in the townhouse complex.
    Therefore, option D effectively undermines the claim made by the tenants' association by highlighting the potential long-term impact of the new policy on a substantial portion of the current tenants.

    Directions: Select a single answer choice.
    Q. Parallelogram OPQR lies in the xy-plane, as shown in the figure below. The coordinates of point P are (2, 4) and the coordinates of point Q are (8, 6). What are the coordinates of point R?
    • a)
      (3, 2)
    • b)
      (3, 3)
    • c)
      (4, 4)
    • d)
      (5, 2)
    • e)
      (6, 2)
    Correct answer is option 'E'. Can you explain this answer?

    Meera Rana answered
    Since OPQR is a parallelogram, line segments PQ and OR have the same length and the same slope. Therefore, in the figure above, PQM and ORN are congruent right triangles. From the coordinates of P and Q, the lengths of the legs of triangle PQM are PM = 8 - 2 = 6 and QM = 6 - 4 = 2. Thus, the lengths of the legs ON and RN of triangle ORN are also 6 and 2, respectively. So the coordinates of point R are (6, 2). The correct answer is Choice E.

    Directions: Answer the questions based on following reading passage.
    Whether the languages of the ancient American peoples were used for expressing abstract universal concepts can be clearly answered in the case of Nahuatl. Nahuatl, like Greek and German, is a language that allows the formation of extensive compounds. By the combination of radicals or semantic elements, single compound words can express complex conceptual relations, often of an abstract universal character[Line 5].
    The tlamatinime (those who know) were able to use this rich stock of abstract terms to express the nuances of their thought. They also availed themselves of other forms of expression with metaphorical meaning, some probably original, some derived from Toltec coinages. Of these forms, the most characteristic in Nahuatl is the juxtaposition of two words that, because they are synonyms, [Line 10] associated terms, or even contraries, complement each other to evoke one single idea. Used metaphorically, the juxtaposed terms connote specific or essential traits of the being they refer to, introducing a mode of poetry as an almost habitual form of expression.
    Q. Which of the following can be inferred from the passage regarding present-day research relating to Nahuatl?
    • a)
      Some record or evidence of the thought of the tlamatinime is available.
    • b)
      For at least some Nahuatl expressions, researchers are able to trace their derivation from another ancient American language.
    • c)
      Researchers believe that in Nahuatl, abstract universal concepts are always expressed metaphorically.
    Correct answer is option 'A,B'. Can you explain this answer?

    Meera Rana answered
    Choice A is correct: the tlamatinime are mentioned in the first sentence of the second paragraph, where it says they were able to use Nahuatl’s stock of abstract terms “to express the nuances of their thought.” This suggests that there is some evidence of what those thoughts were, and therefore Choice A can be inferred.
    Choice B is correct: according to the next sentence, Nahuatl speakers used “forms of expression with metaphorical meaning,” some of which were probably “original” and others “derived from Toltec coinages.” That researchers know certain Nahuatl expressions are derived from Toltec suggests that they are able to trace the derivation of some Nahuatl expressions from another language besides Nahuatl, and therefore Choice B may be inferred.
    Choice C is incorrect: the passage says that in Nahuatl there are single compound words that can express conceptual relations of an “abstract universal character” and mentions “other forms of expression with metaphorical meaning,” but it does not indicate whether metaphorical words or phrases are the only way that abstract universal concepts are expressed in Nahuatl, or whether researchers believe this about Nahuatl. Therefore Choice C cannot be inferred.

    Directions: Compare Quantity A and Quantity B, using additional information centered above the two quantities if such information is given. Select one of the following four answer choices. A symbol that appears more than once in a question has the same meaning throughout the question.
    Q. In the course of an experiment, 95 measurements were recorded, and all of the measurements were integers. The 95 measurements were then grouped into 7 measurement intervals. The graph below shows the frequency distribution of the 95 measurements by measurement interval.
    Quantity A: The average (arithmetic mean) of the 95 measurements
    Quantity B: The median of the 95 measurements
    (d)The relationship cannot be determined from the information given.
    • a)
      Quantity A is greater.
    • b)
      Quantity B is greater.
    • c)
      The two quantities are equal.
    Correct answer is option 'A'. Can you explain this answer?

    Meera Rana answered
    From the histogram, you can observe that
    • all of the measurement intervals are the same size,
    • the distribution has a peak at the measurement interval 6–10, and
    • more of the measurement intervals are to the right of the peak than are to the left of the peak.
    Since in the histogram the 95 measurements have been grouped into intervals, you cannot calculate the exact value of either the average or the median; you must compare them without being able to determine the exact value of either one.
    The median of the 95 measurements is the middle measurement when the measurements are listed in increasing order. The middle measurement is the 48th measurement. From the histogram, you can see that the measurement interval 1–5 contains the first 15 measurements, and the measurement interval 6–10 contains the next 35 measurements (that is, measurements 16 through 50). Therefore, the median is in the measurement interval 6–10 and could be 6, 7, 8, 9, or 10.
    Estimating the average of the 95 measurements is more complicated.
    Since you are asked to compare the average and the median, not necessarily to calculate them, you may ask yourself if you can tell whether the average is greater than or less than the median. Note that visually the measurements in the first three measurement intervals are symmetric around the measurement interval 6–10, so you would expect the average of the measurements in just these three measurement intervals to lie in the 6–10 measurement interval. The 30 measurements in the remaining four measurement intervals are all greater than 10, some significantly greater than 10. Therefore, the average of the 95 measurements is greater than the average of the measurements in the first three measurement intervals, probably greater than 10. At this point it seems likely that the average of the 95 measurements is greater than the median of the 95 measurements. It turns out that this is true.
    To actually show that the average must be greater than 10, you can make the average as small as possible and see if the smallest possible average is greater than 10. To make the average as small as possible, assume that all of the measurements in each interval are as small as possible. That is to say, all 15 measurements in the measurement interval 1–5 are equal to 1, all 35 measurements in the measurement interval 6–10 are equal to 6, etc. Under this assumption, the average of the 95 measurements is
    The value of the smallest possible average, 1,015/95 , is greater than 10.
    Therefore, since the average of the 95 measurements is greater than 10 and the median is in the measurement interval 6–10, it follows that the average is greater than the median, and the correct answer is Choice A.

    The following chart shows the percent distribution of the number of candidates enrolled in a certain test-prep company from 2014 to 2017 for four courses: GMAT, GRE, SAT and LSAT.
    Q. If the total number of candidates increased by 40% from the year 2014 to the year 2017, what is the simple annual percent increase (if necessary whole number rounded) in the number of candidates for the GMAT course between 2014 and 2017?
      Correct answer is '27'. Can you explain this answer?

      Meera Rana answered
      Let the number of candidates in the year 2014 be 100.
      Thus, the number of candidates in the year 2017 = 100 + (100 × 40%) = 140
      Number of candidates for the GMAT course in 2014 = 35% of 100 = 35; read data from the chart
      Number of candidates for the GMAT course in 2017 = 45% of 140 = 63; read data from the chart
      Thus, percent increase = 
      This increase happened over a period of 3 years: 2014-2015, 2015-2016 and 2016-2017.
      Thus, the simple annual percent increase = 80 / 3 = 26.67% ⇒ 27% as whole number rounded
      The answer is not 80% since the question asked to calculate the simple annual percent increase for three years.

      Directions: Answer the questions based on following reading passage.
      When marine organisms called phytoplankton photosynthesize, they absorb carbon dioxide dissolved in seawater, potentially causing a reduction in the concentration of atmospheric carbon dioxide, a gas that contributes to global warming. However, phytoplankton flourish only in surface waters where iron levels are sufficiently high. Martin therefore hypothesized that adding iron to iron-poor regions of the ocean could help alleviate global warming. While experiments subsequently confirmed that such a procedure increases phytoplankton growth, field tests have shown that such growth does not significantly lower atmospheric carbon dioxide. When phytoplankton utilize carbon dioxide for photosynthesis, the carbon becomes a building block for organic matter, but the carbon leaks back into the atmosphere when predators consume the phytoplankton and respire carbon dioxide.
      Q. It can be inferred from the passage that Martin’s hypothesis includes which of the following elements?
      [For the following question, consider each of the choices separately and select all that
      apply.]
      • a)
        A correct understanding of how phytoplankton photosynthesis utilizes carbon dioxide
      • b)
        A correct prediction about how the addition of iron to iron-poor waters would affect phytoplankton growth
      • c)
        An incorrect prediction about how phytoplankton growth would affect the concentration of atmospheric carbon dioxide
      • d)
        None of these
      Correct answer is option 'A,B,C'. Can you explain this answer?

      All three choices are correct.
      Martin’s hypothesis was that adding iron to iron-poor regions of the ocean could help alleviate global warming.
      Choice A is correct: the passage presents Martin as using the standard understanding of how phytoplankton photosynthesize as a basis for the hypothesis.
      Choice B is correct: the passage states that experiments confirmed that adding iron to iron-poor regions increases phytoplankton growth in those regions. Therefore, Martin’s prediction about this was correct.
      Choice C is correct: it can be inferred that in Martin’s hypothesis the means by which adding iron in certain regions could alleviate global warming is that phytoplankton increase in those regions and absorb atmospheric carbon dioxide. The passage states that predators who consume phytoplankton respire carbon dioxide, so that the carbon dioxide absorbed by phytoplankton reenters the atmosphere. Therefore, Martin’s prediction about this was incorrect.

      Direction: Read the following passage carefully and answer the questions given below it. 
      According to recent research, during the final days of planet formation but before planets had fully formed, planetesimals, which were itinerant objects as large as Pluto, may have collided with Earth, Mars, and the moon. It is hypothesized that they deposited siderophiles (elements such as gold, platinum, and palladium) into Earth’s crust.
      Siderophiles are typically drawn to iron, which forms much of the inner core of Earth. Logically, during planet formation, most of these elements would have been drawn into the center of Earth; however, their abundance in Earth’s crust has long perplexed scientists. These collisions may also have caused Earth’s axis to tilt by 10 degrees and deposited water on the moon.
      Q. Based on the information in the passage, which of the following best describes planetesimals?
      • a)
        Free-roaming proto-planets
      • b)
        Planets that collided with Earth and bounced out of the galaxy
      • c)
        Objects embedded in Earth’s crust
      • d)
        Highly magnetic bodies that helped form planets
      • e)
        Large rocks composed of siderophiles
      Correct answer is option 'A'. Can you explain this answer?

      Though it is not explicitly stated in the passage, you can, from a few clues, determine that a planetesimal was (1) an object that moved—a sort of wandering body—since they are described as “itinerant,” and (2) an object that existed during “planet formation.” Choice (A) follows from those two inferences and is correct. There is no support for choice (B); the passage makes no mention of objects leaving or bouncing out of the galaxy. You can eliminate choice (C) because the passage tells you that components of the planetesimals (the siderophiles) remained in Earth’s crust aer impact but doesn’t say that whole planetesimals did so.
      Choice (D) is beyond the scope of the passage, which doesn’t contain information about planetesimals’ magnetic characteristics. You can also cross off (E) because the passage doesn’t describe planetesimals as “rocks,” nor does it claim that they are always made up of (or necessarily even contain) siderophiles.

      Directions: For the following question, select all the answer choices that apply.
      Q. The integer v is greater than 1. If v is the square of an integer, which of the following numbers must also be the square of an integer?
      Indicate all such numbers.
      • a)
        81v
      • b)
        25v +10 √v +1
      • c)
        4v2 + 4 √v +1
      Correct answer is option 'A,B'. Can you explain this answer?

      Meera Rana answered
      If v is the square of an integer, then √v is an integer. You can use this fact, together with the fact that the product and the sum of integers are also integers, to examine the first two choices.
      Choice A: The positive square root of 81v is 9 √v, which is an integer. So 81v is the square of an integer.
      Choice B: 25v + 10 √v + 1 = (5 √v + 1)2 and 5√v + 1 is an integer. So 25v + 10v + 1 is the square of an integer.
      Choice C: Since there is no obvious way to factor the given expression, you may suspect that it is not the square of an integer. To show that a given statement is not true, it is sufficient to find one counterexample. In this case, you need to find one value of v such that v is the square of an integer but 4v2 + 4√v + 1 is not the square of an integer. If v = 4, then 4v2 + 4√v + 1 = 64 + 8 + 1 = 73, which is not the square of an integer. This proves that 4v2 + 4√v + 1 does not have to be the square of an integer.
      The correct answer consists of Choices A and B.

      I was used to David’s scheming, but even I was _____ of the outrageous boldness of his latest plan.
      • a)
        Nonplussed
      • b)
        Incredulous
      • c)
        Victimized
      • d)
        Disbelieving
      • e)
        Forthright
      Correct answer is option 'B,D'. Can you explain this answer?

      Meera Rana answered
      The author states he/she was used to David's scheming, and the phrase "but even I was---" indicates that the author managed to be shocked by his latest plan. Therefore, the correct answers are B) incredulous and D) disbelieving. Nonplussed, which means unimpressed, does not fit the context of the sentence. Victimized, which means to be singled out for cruel or unjust treatment, might fit the context of the sentence, but there is no other choice with the same meaning. Forthright, which means candid, does not make sense. Finessed, which means to have brought about by skilful maneuvering, might fit the context of the sentence, but there is no other choice with the same meaning.

      Directions: After reading the passage, read and respond to the questions that follow by selecting the best choice for each one.
      Statins are a class of drugs that work to decrease the level of cholesterol in the blood. They are able to perform this function by effectively blocking the cholesterol-producing enzyme in the liver. In recent years, statins have increased in utility and popularity. Although a certain amount of cholesterol is imperative for the human body to function, an elevated level can cause a precarious situation in the body. Cholesterol affixes itself to arteries, lining them and inhibiting blood flow. Since less blood can travel through those encumbered arteries, the blood flow to the heart is adversely affected. Scientists have found that the walls of arteries in the body can become inflamed from this plaque buildup. In many instances, the end result of this decreased blood supply and inflammation is a heart attack sometimes a fatal one. In other cases the blood supply to the brain is compromised by the plaque buildup, often causing a stroke. Decreased blood flow to the legs can cause leg pains or cramps. In short, arterial plaque buildup is never a healthy situation.
      Statins diminish the amount of cholesterol generated by the body. Although the drugs are not always able to reduce the amount of plaque that may already be clogging arteries, they can slow the production of new plaque. The drugs are also able to stabilize the plaques that are already present and make them less likely to cause problems in the body.
      Lowered cholesterol does not guarantee that a heart attack won’t occur, but statin use will lower the risk for most patients. Not everyone who has a heart attack has high cholesterol levels, but most do have plaque formations on their arteries. It should be noted that the plaque is not always formed by high levels of cholesterol in the blood.
      Statins are generally prescribed by doctors for people with elevated cholesterol levels. As the mean weight of the American population has risen, so has the number of individuals with high cholesterol. Millions of men and women in this country are prescribed statins in an effort to decrease the amount of cholesterol in their blood. Remarkably, these effects can be seen in as little as two weeks after beginning a statin regime.
      Q. Which of the following is suggested about the American population?
      • a)
        Their life expectancy has decreased in the past few decades.
      • b)
        They do not get enough exercise
      • c)
        Their diet may be affecting their cholesterol levels.
      • d)
        They want an easy fix for a difficult problem.
      • e)
        There are a record number of heart disease deaths in the United States.
      Correct answer is option 'C'. Can you explain this answer?

      The correct answer is (C). The article says that the average weight of Americans has risen and so have their cholesterol levels. Higher levels are partially due to improper eating habits. The other responses can not be inferred from what is presented in the article.

      Directions: Answer the questions based on following reading passage.
      At a certain period in Earth’s history, its atmosphere contained almost no oxygen, although plants were producing vast quantities of oxygen. As a way of reconciling these two facts, scientists have hypothesized that nearly all of the oxygen being produced was taken up by iron on Earth’s surface. Clearly, however, this explanation is inadequate. New studies show that the amount of iron on Earth’s surface was not sufficient to absorb anywhere near as much oxygen as was being produced. Therefore, something in addition to the iron on Earth’s surface must have absorbed much of the oxygen produced by plant life.
      Q. In the argument given, the two portions in boldface play which of the following
      roles?
      • a)
        The first is a claim made by the argument in support of a certain position; the second is that position.
      • b)
        The first is a judgment made by the argument about a certain explanation; the second is that explanation.
      • c)
        The first expresses the argument’s dismissal of an objection to the position it seeks to establish; the second is that position.
      • d)
        The first sums up the argument’s position with regard to a certain hypothesis; the second provides grounds for that position.
      • e)
        The first is a concession by the argument that its initial formulation of the position it seeks to establish requires modification; the second presents that position in a modified form.
      Correct answer is option 'D'. Can you explain this answer?

      Meera Rana answered
      The passage presents an argument and the question asks you to identify the role the portions highlighted in boldface play in that argument. The first step in responding is to read through the passage quickly to get an understanding of what is being said. Then it is possible to go back and assess how the parts of the passage fit together into an argument.
      In this passage the first sentence presents two pieces of information that seem to be in conflict—the atmosphere contained almost no oxygen even though plants were producing so much of it. The second sentence presents a hypothetical explanation that has been proposed for reconciling the discrepancy — that oxygen was absorbed by iron. The next sentence calls this hypothetical explanation inadequate and the following sentence gives a reason for that judgment — that there was insufficient iron for the proposed explanation to work. Finally, the last sentence draws the conclusion that there must have been something in addition to iron to absorb the oxygen.
      Since the highlighted portions in the passage represent the main content of the third and fourth sentences, the task in this question is to find the answer choice whose two parts fit those sentences’ roles. It can be seen that answer Choice D fits the requirement: the third sentence does sum up the argument’s position about a hypothesis, and the fourth sentence gives grounds for the third. Therefore Choice D is the correct answer.

      Direction: Read the following passage carefully and answer the questions given below it. 
      Upton Sinclair’s 1906 novel The Jungle, set in the meatpacking plants of early 20th-century Chicago, was intended to provoke a strong public reaction. The book, based on Sinclair’s own experience working incognito at a meatpacking plant, tells of the horrific working conditions in the meat industry. The stories of contaminated or diseased meat were intended to incite outrage at the system of worker exploitation that led to these issues; however, the response of both the public and the government focused on food safety. Less than a year after publication of The Jungle, Congress passed the Pure Food and Drug Act, which eventually led to the creation of the modern Food and Drug Administration. Sinclair himself was not pleased with what he saw as a misinterpretation of his work; he famously said, “I aimed at the public's heart, and by accident I hit it in the stomach.”
      The reaction to The Jungle is hardly the first time that concerns over food safety and purity have won out over more compassionate objectives. In 1516, Bavaria (then part of the Holy Roman Empire) passed the Reinheitsgebot, a law mandating, among other things, that the only ingredients that could be used in the production of beer were barley, hops, and water. Although Reinheitsgebot literally translates to “purity order,” the main impetus behind the law was not to protect the purity of beer but rather to prevent competition between bakers and brewers for the purchase of grains. By ensuring that wheat and rye would be available only for bakers, the Bavarian government hoped to keep bread affordable for everyone. Many modern breweries, however, tout that their beers conform to the Reinheitsgebot in an attempt to convince consumers of the quality and purity of their beer, completely ignoring the original empathetic intent of the law. Perhaps the public’s stomach is, in fact, more sensitive than its heart.
      Q. With which of the following statements about the Pure Food and Drug Act would the author of the passage likely agree?
      • a)
        It did not do enough to ensure the safety of America’s food supplies, given the state of the meatpacking industry.
      • b)
        It was a misguided response to the transgressions of the meatpacking industry exposed in The Jungle.
      • c)
        Congress enacted the law to limit competition for resources, although its effect was to improve food quality.
      • d)
        The act was likely to increase the price of food by imposing regulations on the meatpacking industry.
      • e)
        It was intended to fix problems that were not highlighted in The Jungle.
      Correct answer is option 'B'. Can you explain this answer?

      Meera Rana answered
      The question asks what the author of the passage would think about the Pure Food and Drug Act; this law is discussed in the first paragraph, so start your research there. The only opinion there is that of Upton Sinclair, who was not happy that the act focused on food safety rather than the worker exploitation he sought to highlight in The Jungle. Look further to find the opinion of the passage's author. In the first sentence of the second paragraph, the author indicates an opinion through a comparison, saying that "concerns over food safety and purity . . . won out over more compassionate objectives." Thus, the author feels that Congress with the Pure Food and Drug Act addressed a less important problem than it should have. Then again, in the last sentence of the passage, the author expresses agreement with Sinclair, saying that “the public’s stomach is, in fact, more sensitive than its heart.” Look for an answer choice that lines up with the opinion. (B) does so nicely; the author would think, just as Sinclair did, that this law was a misguided response to The Jungle, so this is the correct answer.
      (A) is incorrect because the author does not express any opinion on the effectiveness of the law; she thinks a law protecting workers would have been a more appropriate response, but that doesn’t mean she thinks the law that was passed didn’t work. (C) confuses the U.S. law this question asks about with the Bavarian law discussed in the second paragraph. (D) is wrong because the only discussion of food prices comes in the second paragraph in connection to the Reinheitsgebot, not the Pure Food and Drug Act. (E) may be tempting because the author does think the act addressed a problem that should have been a lower priority. However, it's clear that The Jungle did highlight a problem with food quality, given its "stories of contaminated or diseased meat," and the law addressed that issue.

      Directions: Choose the letter that best answers the question.
      Q. The perimeter of triangle BCD is 18. What is its area?
      • a)
        8
      • b)
        10
      • c)
        20
      • d)
        12
      • e)
        24
      Correct answer is option 'D'. Can you explain this answer?

      Meera Rana answered
      The correct answer is (D). To solve, first find the length of side BD. We know that the perimeter is 14, so by subtracting the total of the sides we know, we find that this side is 3. This tells us that it is an isosceles triangle, with two equal sides and two equal angles. To find the area, we use the formula 1/2bh.
      We know that the base is 8. To find the height, draw a line from point D straight down to cut line BC in half. Name the point where the line hits BC as E. This new line forms two right triangles, CED and BED, each with a side of 3 and a hypotenuse of 5. We can tell these are 3-4-5 right triangles. The height of the original triangle is 5. We can then solve the equation to find the area:

      In some countries tipping waiters is considered a norm whereas in some others, it is believed to be an insult, making tipping a __________ issue.
      [ Note: Select 2 answer choices ]
      • a)
        contentious
      • b)
        treacherous
      • c)
        irrefutable
      • d)
        perfidious
      • e)
        controversial
      Correct answer is option 'A,E'. Can you explain this answer?

      Alok Saxena answered
      Introduction:
      Tipping is a practice that varies across different countries and cultures. While it is considered a norm in some countries, it is seen as insulting in others. This makes tipping a contentious and controversial issue.

      Contentious:
      Tipping can be described as contentious because it generates a lot of disagreement and controversy. Different people have contrasting views on whether or not to tip, how much to tip, and who to tip. The practice of tipping can often lead to heated debates and arguments, as there is no universally accepted standard for tipping.

      Controversial:
      Tipping is also controversial because it raises questions about fairness and equity. Some argue that tipping allows customers to reward good service, while others believe it creates an unequal system where waiters depend on tips to make a living wage. The controversy surrounding tipping has led to debates about whether it should be abolished or replaced with fairer alternatives, such as higher wages for service industry workers.

      Cultural Differences:
      The difference in tipping norms across countries is largely influenced by cultural factors. In some cultures, tipping is seen as a way to show gratitude and appreciation for good service. In these countries, not tipping may be considered rude or disrespectful. On the other hand, in cultures where tipping is not customary, leaving a tip may be seen as an insult or implying that the service was inadequate.

      Examples:
      - In the United States, tipping is considered a norm and is expected in most establishments. It is customary to tip waiters around 15-20% of the total bill.
      - In Japan, tipping is not a common practice and may even be seen as rude. The Japanese believe that good service is the standard and should not be rewarded with extra money.
      - In France, a service charge is typically included in the bill, so tipping is not necessary. However, leaving a small amount of change as a gesture of appreciation is common.

      Conclusion:
      The practice of tipping waiters varies across different countries and cultures, making it a contentious and controversial issue. The differing views on tipping reflect cultural norms and beliefs about fairness and gratitude. Understanding these cultural differences is important when traveling to different countries to avoid unintentionally insulting or disrespecting waitstaff.

      Each side of a cuboid is integer and their sum is 6 units.
      Quantity A: Total surface area of the cuboid
      Quantity B: 25
      • a)
        Quantity A is greater
      • b)
        Quantity B is greater
      • c)
        The two quantities are equal
      • d)
        The relationship cannot be determined from the information given
      Correct answer is option 'B'. Can you explain this answer?

      We know that there are three sides of a cuboid. Say the sides are a,b & c unit.
      Thus, the total surface area of the cuboid = 2(ab + bc + ca).
      We have to compare the value of 2(ab + bc + ca) against 25 such that (a + b + c) = 6 and a, b & c are integers.
      Let's try to compute the maximum possible value of 2(ab + bc + ca); if the computed value of 2(ab + bc + ca) <  25, we have an answer as option B; however, if the computed value of 2(ab + bc + ca)>25, then we must compute the minimum possible value of 2(ab + bc + ca) too. If the minimum possible value of 2(ab + bc + ca) is also greater than 25, then we have an answer as option A. Else the answer is Option D.
      Note that if the sum of three numbers is constant, here 6, the product of the numbers would be maximum only if the numbers are equal.
      ⇒ The product of a,b & c would be maximum if a = b = c = 63 = 2.
      ⇒ The maximum possible value of 2(ab + bc + ca)
      =2(2 × 2 + 2 × 2 + 2 × 2) = 24
      We see that Quantity B (= 25) is greater than Quantity A (= 24).
      There can be two more combinations of the sides of the cuboid:
      a = 1;b = 2;c = 3
      Needless to state that the value of 2(ab+bc+ca) would be less than 24.
      Let's compute that.
      2(ab + bc + ca) = 2(1 × 2 + 2 × 3 + 1 × 3) = 22 < 24
      a = 1;b = 1;c = 4
      Needless to state that the value of 2(ab+bc+ca) would be less than 24.
      Let's compute that.
      2(ab + bc + ca) = 2(1 × 1 + 1 × 4 + 1 × 4) = 18 < 24
      The correct answer is option B.

      Direction: Read the following passage carefully and answer the questions given below it.
      Characterized as half zebra and half horse, the quagga sounds like a mythical creature, but at one time, it was a very real animal. Only 150 years ago, a great number of quaggas were found in South Africa.
      Unlike the common zebra, which has black and white stripes that cover its entire body, the quagga has yellow-brown stripes only on its head, neck, and forebody, which gave it its half-horse facade. Its unique appearance caused early explorers to think the quagga was a separate species from the common zebra when it was discovered around 1760, but DNA analysis later revealed that the animal is a subspecies of the zebra. In the 100 years following its discovery, the quagga population diminished. It was frequently hunted for its meat and hide, and settlers who considered the animal a competitor for the grazing of their livestock also callously killed the quagga in great numbers. By 1870, the quagga was no longer found in the wild. The last captive quagga died in 1880 in an Amsterdam zoo. Today, the quagga is categorized as an extinct animal, but researchers in Africa hope to resurrect the native subspecies through genetic modification and selective breeding.
      Q. With which of the following statements would the author of the passage most likely agree?
      • a)
        The zebra is a type of quagga.
      • b)
        Researchers hope to restore the quagga using scientific technology.
      • c)
        Settlers humanely killed quaggas in an effort to protect their livestock.
      • d)
        Shortly after the quagga was discovered, its population grew.
      • e)
        Settlers saw the quagga as a competitor for the prey of their livestock.
      Correct answer is option 'B'. Can you explain this answer?

      Meera Rana answered
      The correct answer may not be stated explicitly in the passage, but it must follow directly from something that is stated there. The passage states that the quagga is a subspecies of the zebra, not the other way around. That makes choice (A) incorrect. The passage also contradicts choice (D); its population “diminished” aer discovery. Choice (C), which states that settlers “humanely” killed quaggas, contradicts the passage more subtly but just as fatally. In fact, according to the passage, settlers “callously” killed quaggas. Choice (E) distorts the passage. The quagga was a competitor for the pasture of settlers’ livestock, not their prey.
      Indeed, grazing animals have no prey. Choice (B) is the correct answer, as the last sentence of the passage states that scientists are hoping to spark the species’ return.

      Six months after Hurricane Maria, the island __________ with the longest blackout in the U.S. history.
      [ Note: Select 2 answer choices ]
      • a)
        skirmishes
      • b)
        dissents
      • c)
        grapples
      • d)
        brawls
      • e)
        wrestles
      Correct answer is option 'C,E'. Can you explain this answer?

      Aryan Sharma answered
      Six months after Hurricane Maria, the island __________ with the longest blackout in the U.S. history.

      Explanation:
      The question is asking for two answer choices that describe the situation on the island six months after Hurricane Maria, specifically relating to the longest blackout in U.S. history.

      Key Points:
      - Hurricane Maria: A powerful Category 4 hurricane that struck Puerto Rico in September 2017, causing widespread destruction and devastation.
      - Blackout: The loss of electricity supply to an area, resulting in a lack of power for an extended period.

      Answer Explanation:
      The correct answer choices are 'C' and 'E': grapples and wrestles.

      Grapples:
      - The term "grapples" implies that the island is struggling or grappling with the aftermath of the hurricane and the resulting blackout.
      - It suggests that the situation is challenging and that the island is actively trying to overcome the difficulties it is facing.

      Wrestles:
      - The term "wrestles" also suggests that the island is engaged in a struggle or fight against the blackout.
      - It conveys the idea that the recovery process is ongoing and that the island is actively working to restore power and infrastructure.

      Overall:
      - Both "grapples" and "wrestles" accurately describe the situation on the island six months after Hurricane Maria.
      - The island is facing significant challenges and is actively working to recover from the devastating effects of the hurricane, including the longest blackout in U.S. history.

      Directions: Answer the questions based on following reading passage.
      Elements of the Philosophy of Newton, published by Voltaire in 1738, was an early attempt to popularize the scientific ideas of Isaac Newton. In the book’s frontispiece, Voltaire is seen writing at his desk, and over him a shaft of light from heaven, the light of truth, passes through Newton to Voltaire’s collaborator Madame du Châtelet; she reflects that light onto the inspired Voltaire. Voltaire’s book commanded a wide audience, according to Feingold, because “he was neither a mathematician nor a physicist, but a literary giant aloof from the academic disputes over Newtonian ideas.” In other words, Voltaire’s amateurism in science “was a source of his contemporary appeal, demonstrating for the first time the accessibility of Newton’s ideas to nonspecialists.”
      Q. Which of the following statements about Voltaire’s Elements of the Philosophy of
      Newton can be inferred from the passage?
      [For the following question, consider each of the choices separately and select all that
      apply.]
      • a)
        Voltaire’s literary stature helped secure a large audience for this attempt to popularize Newton’s ideas.
      • b)
        Voltaire’s status as a nonscientist was an advantage in this effort to bring Newtonian science to the attention of the general public.
      • c)
        The frontispiece of the book implies that Voltaire’s understanding of Newton’s ideas was not achieved without assistance.
      • d)
        None of these
      Correct answer is option 'A,B,C'. Can you explain this answer?

      All three choices are correct.
      Choice A is correct: the paragraph states that one of the reasons Voltaire’s book commanded a wide audience is that he was “a literary giant.”
      Choice B is correct: the paragraph states that Voltaire’s amateurism in science demonstrated that nonspecialists could also understand Newton’s ideas.
      Choice C is correct: the paragraph refers to Voltaire’s collaborator, Madame du Châtelet. In the image described, she serves as the intermediary between Newton and Voltaire, conveying Newton’s ideas to Voltaire.

      Directions: After reading the passage, read and respond to the questions that follow by selecting the best choice for each one.
      (1) Max Weber aimed to make sociology objective and systematic, like the natural sciences. This was a challenge since historical and social reality, the subject matter of sociology, is inherently chaotic and dependent on individual - possibly idiosyncratic or irrational – events and on the subjective interpretations of those events by the contemporary and the subsequent societies and by historians and sociologists. The research and analysis methods of natural scientists would not have worked in sociology. Therefore, he created a conceptual tool called the pure-type.
      (2) The pure-type is formed by synthesizing the characteristics and elements of many real-world manifestations of a particular phenomenon into a unified analytical construct that defines the general and logically-consistent features of that phenomenon. For example, Weber studied capitalism and integrated the distinctive features of this economic system across cultures and time-periods to create a pure-type. According to this pure-type, the attributes of capitalism are – pursuit of profit as the sole objective, private ownership of all potential means of profit, competition between companies, and a laissez faire government that does not interfere with the free markets.
      (3) A pure-type is not meant to describe all the characteristics of any one instance of a phenomenon. Moreover, no single instance fully adheres to the pure-type. The present-day United States is regarded as the most capitalist nation in world history; however, it too deviates significantly from Weber's pure-type of capitalism. For example, the country has many markets that are dominated by a single large company and hence, are not competitive. The value of a pure-type of capitalism is that it defines the features on which the economic systems of various countries may be objectively compared and on which the changes in the economic system of a country over time may be objectively measured.
      Q. For the following question, consider each of the choices separately and select all that apply.
      It can be inferred from the passage that a country that fully adhered to Weber's pure-type of capitalism would have which of the following characteristics?
      [ Note: Select one or more answer choices ]
      • a)
        Regulation of free markets by government to prevent injustices against the poor
      • b)
        No individuals or companies who work in pursuit of objectives other than profit
      • c)
        No market dominated by a single large company
      Correct answer is option 'B,C'. Can you explain this answer?

      Meera Rana answered
      The passage says that no actual instance of a phenomenon fully adheres to the pure-type of that phenomenon. Therefore, no actual country is likely to fully adhere to Weber's P-T of capitalism. The country mentioned in this question is likely to a hypothetical country.
      Since this country fully adheres to this P-T, it will fully satisfy (100%, no exceptions at all) all four attributes mentioned in this P-T:
      • pursuit of profit as the sole objective
      • private ownership of all potential means of profit
      • competition between companies
      • a laissez faire government that does not interfere with the free markets.
      Let us analyze the options one by one.
      A. This option is incorrect. Regulation by government of free markets for any reason violates the fourth attribute of the P-T. A country that fully adheres to all four attributes of the P-T, therefore, will not have this characteristic.
      B. This option is correct. In a country that fully adhered to the first attribute of the P-T, profit would be the sole objective of everyone.
      C. This option is correct. The example of present-day United States in the passage includes this sentence: "the country has many markets that are dominated by a single large company and hence, are not competitive." Therefore, for a market to be competitive, it should not be dominated by a single large company. For a country that 100% satisfies the third attribute of the P-T, no market would be dominated by a single large company.
      The correct answer are B and C.

      Directions: Refer to the following passage. After reading the passage, read and respond to each question selecting the best answer choice for each one.
      For hot desert locations with access to seawater, a new greenhouse design generates freshwater and cool air. Oriented to the prevailing wind, the front wall of perforated cardboard, moistened by a trickle of seawater pumped in, cools and moistens hot air blowing in. This cool, humidified air accelerates plant growth; little water evaporates from leaves. Though greenhouses normally capture the heat of sunlight, a double-layered roof, the inner layer coated to reflect infrared light outward, allows visible sunlight in but traps solar heat between the two layers. This heated air, drawn down from the roof, then mixes with the greenhouse air as it reaches a second seawater-moistened cardboard wall at the back of the greenhouse. There the air absorbs more moisture before being cooled off again when it meets a seawater-cooled metal wall, which causes moisture in the air to condense. Thus distilled water for irrigating the plants collects.
      Q. It can be inferred that the process described in the passage makes use of which of the following?
      [For the following question, consider each of the choices separately and select all that apply.]
      • a)
        The tendency of hot air to rise
      • b)
        The directional movement of wind
      • c)
        Variation in the moisture capacity of air
      Correct answer is option 'B,C'. Can you explain this answer?

      Choices B and C are correct. This question asks the reader which of the three phenomena listed in the answer choices is used in the process described in the passage.
      Choice A is incorrect: the passage does not indicate that the tendency of hot air to rise is used in the process, and in fact says that heated air is drawn down, not up, as part of the greenhouse design.
      Choice B is correct: the second sentence describes the orientation of a perforated cardboard wall toward the prevailing wind so that hot air blows in and is moistened.
      Choice C is correct: the process depends on the ability of hot air to contain moisture that is then deposited when the air cools.

      Direction: For each sentence, choose one word for each set of blanks. Select the word or words that best fit(s) the meaning of the sentence as a whole
      As a society grows and evolves, so too do its cultural norms. A piece of art or a musical performance that once was viewed as inordinately (i) _____________ may now be seen as utterly (ii) ___________
      • a)
        A,F
      • b)
        B,E
      • c)
        A,D
      • d)
        C,D
      • e)
        C,F
      Correct answer is option 'A'. Can you explain this answer?

      Meera Rana answered
      There are no obvious road signs here, so read the sentence carefully for clues to the blanks. The first sentence explains that cultural norms, or what a society views as acceptable, evolve over time. The two blanks describe two different views of the same piece of art, and each blank describes a different extreme, as indicated by “inordinately” (“exceeding limits”) and “utterly” (“to the highest degree”). Note that there’s no way to tell which way the views evolved; they may have gone from acceptable to unacceptable, or vice versa. You’ll have to consider both blanks to find words with the appropriate relationship. The only words that fit are (A) salacious, which means “morally offensive” or “indecent,” and (F) pedestrian, which in this context means “commonplace” or “mundane.” Taken together, they describe how a work of art can at one point be seen as shocking but later be seen as ordinary.
      (B) adequate doesn’t fit with “inordinately.” It’s not possible to be extremely adequate—something either is or is not adequate. (C) trite means “boring” or “unoriginal.” This could potentially fit as one part of the comparison, but there’s no appropriately contrasting word for the second blank. (D) pensive means “thoughtful,” and (E) opulent means “lavish” or “luxurious”; neither of these fits into the context of art and cultural norms, and neither contrasts with a choice for the first blank.

      Length Of Unemployment for Workers in Region X for Two Industries, 2003
      Q. In the circle graphs, the degree measure of the central angle of the sector representing the number of workers unemployed for 11 to 14 weeks is how much greater in the manufacturing industry graph than in the service industry graph?
      • a)
      • b)
        20°
      • c)
        10°
      • d)
        18°
      • e)
        15°
      Correct answer is option 'D'. Can you explain this answer?

      Meera Rana answered
      Recall that in a circle graph, the degree measure of the central angle of a sector representing n percent of the data is equal to n percent of 360°.
      The degree measure of the sector representing the number of workers unemployed for 11 to 14 weeks is 10% of 360°, or 36°, for the manufacturing industry graph and is 5% of 360°, or 18°, for the service industry graph. Since 36° − 18° = 18°, the measure of the central angle of that sector in the manufacturing industry graph is 18° greater than the measure of the central angle of the corresponding sector in the service industry graph. The correct answer is Choice b.

      Directions: Questions are based on the following data. For these questions, select a single answer choice unless otherwise directed.
      Q. In the winter, 2/3 of the cardinal sightings, 1/2 of the junco sightings, and 1/4 of the sparrow sightings were in January. What fraction of the total number of sightings of these three bird species in the winter were in January?
      • a)
        1/4
      • b)
        1/3
      • c)
        1/2
      • d)
        2/3
      • e)
        3/4
      Correct answer is option 'C'. Can you explain this answer?

      Meera Rana answered
      The total number of sightings of cardinals, juncos, and sparrows in the winter was 30 + 12 + 20, or 62. In the question, you are given that 2/3 of the 30 cardinal sightings, ½ of the 12 junco sightings, and ¼ of the 20 sparrow sightings were in January. Therefore the number of sightings of these three bird species in January was , or 31, which accounted for 31/62 or ½, of the total number of sightings of these three bird species in the winter. Thus the correct answer is Choice C.

      The speed of light is approximately 3.00 x 108 meters/sec.
      In scientific notation how many kilometers per hour is the speed of light?
      • a)
        1.08 x 1012
      • b)
        3.00 x 106
      • c)
        8.33 x 104
      • d)
        1.08 x 109
      Correct answer is option 'D'. Can you explain this answer?

      To convert the speed of light from meters per second to kilometers per hour, we need to perform the following steps:

      Step 1: Convert meters to kilometers
      To convert meters to kilometers, we divide the value by 1000 since there are 1000 meters in a kilometer.

      Speed of light in kilometers per second = 3.00 x 10^8 meters/second ÷ 1000 = 3.00 x 10^5 kilometers/second

      Step 2: Convert seconds to hours
      To convert seconds to hours, we need to multiply the value by 3600 since there are 3600 seconds in an hour.

      Speed of light in kilometers per hour = 3.00 x 10^5 kilometers/second x 3600 = 1.08 x 10^9 kilometers/hour

      Therefore, the speed of light in kilometers per hour is approximately 1.08 x 10^9 kilometers/hour, which corresponds to option D.

      Direction: Read the following passage carefully and answer the questions given below it.
      Cinematic renditions of historic pieces of literature provide an informative glimpse into the cultural and social context in which the films were made. Shakespeare’s Henry V is a prime example, as it has been in circulation within the English-speaking world for over 400 years and has been reinterpreted in a number of different milieus. Since the source material has not changed, the way in which different artists and directors treat the play indicates not only the predispositions of the interpreter, but also the prevailing social and political views of the audience. This is acutely noticeable in a play like Henry V, which is highly charged with nationalistic concerns.
      The play was written during the reign of Elizabeth I, when English national identity (and the modern English language) had begun to crystallize and the language and culture we know today approached their present form. It is a historical biography of King Henry V of England, who waged a bloody campaign during The Hundred Years War with the aim of conquering France. The introduction of the play features an adviser to the King explaining, in a confusing and nearly incomprehensible fashion, the justification for Henry’s claim to the French throne. The text of the play itself has been interpreted as being ambiguous in its treatment of Henry’s character. Henry has a number of rousing, heroic speeches, but he is also shown to be coldly unmerciful, as in the case of his refusal to pardon petty thieves.
      Shakespeare’s play has been adapted in two famous film versions. The first, directed by Laurence Olivier, was made during the Second World War, immediately before the invasion of Normandy was launched in 1944. Critics of the film have emphasized the pageantry, bravado, and nationalistic undertones of this version. The battle scenes in the film are understated and tame, with little of the carnage that would be expected of a medieval melee. They are shot in beautiful weather, and the actors are clad in radiant colors. The scene with Henry’s harsh justice is omitted. The film was funded, in part, by the British government and is widely understood to have been intended as a propaganda film, made in anticipation of D-day. The second version, directed by Kenneth Branagh, was made in 1989, only a few years after the Falklands War, and was much harsher in tone. The battle scenes are gory and are shot in gray, dismal weather. The actors wear muddy, blood-smeared costumes reflective of the period. The scene with Henry’s harsh justice is included.
      Q. Which of the following most accurately describes the relationship between the highlighted sentences?
      • a)
        The first is an example of an argument; the second is a counterexample.
      • b)
        The first is a synthesis of disparate ideas; the second is one of the components of that synthesis.
      • c)
        The first is the topic of the passage; the second is an argument in support of it.
      • d)
        The first presents an assertion; the second provides an example to support that assertion.
      • e)
        The first is a thesis; the second is the antithesis.
      Correct answer is option 'D'. Can you explain this answer?

      Meera Rana answered
      In this case, the first highlighted sentence is an assertion that lays out the topic and scope of the passage. The second highlighted sentence is a specific example that the author offers to illustrate his main point. Choice (D), the correct answer, hits this prediction squarely.
      Choice (A) is incorrect because the second sentence does not contradict the first. You can reject (B) as it mischaracterizes the first sentence; there is nothing disparate about its components. You can reject (C) because it gets the second sentence wrong; the second sentence is an example, not an argument (which would need a conclusion supported by evidence).
      Choice (E) distorts the relationship between the two sentences; the second doesn’t contradict the first.

      Directions: Refer to the following passage. After reading the passage, read and respond to each question selecting the best answer choice for each one.
      I enjoyed A Dream of Light & Shadow: Portraits of Latin American Women Writers for the same reasons that, as a child, I avidly consumed women’s biographies: the fascination with how the biographical details of another female’s life are represented and interpreted.
      A Dream offers a rich read, varied in both the lives and texts of the women portrayed, and the perspectives and styles of the sixteen essayists. Yet, as an adult, I have come to demand of any really “great” book a self-consciousness about the tenuous nature of representations of reality, a critical contextualization of florid detail, and a self-awareness of the role of ideology in our lives. In these critical senses, A Dream is inadequate.
      Q. The author of the passage suggests that A Dream falls short in which of the following respects?
      • a)
        It does not appear to recognize that representations of reality can be unreliable.
      • b)
        It seems to focus on stylistic variety at the expense of accuracy of detail.
      • c)
        It offers a wealth of detail without sufficient critical examination of that detail.
      Correct answer is option 'A,C'. Can you explain this answer?

      Meera Rana answered
      Choices A and C are correct. We know from the final sentence that the collection falls short of several criteria established by the author.
      Choice A is correct: the book does not demonstrate sufficient awareness of the “tenuous nature of representations of reality.”
      Choice B is incorrect: there is no mention in the passage of any concern on the part of the author about the accuracy of detail.
      Choice C is correct: the book does not offer an adequate “critical contextualization of florid detail.”

      Directions: Select a single answer choice.
      Q. Of the 40 specimens of bacteria in a dish, 3 specimens have a certain trait. If 5 specimens are to be selected from the dish at random and without replacement, which of the following represents the probability that only 1 of the 5 specimens selected will have the trait?
      • a)
      • b)
      • c)
      • d)
      • e)
      Correct answer is option 'E'. Can you explain this answer?

      In the context of this problem represents the number of ways r specimens can be selected without replacement from n specimens.
      The probability that only 1 of the 5 specimens selected from the 40 specimens will have the trait is equal to
      number of ways to select 5 specimens, only 1 of which has the trait / number of ways to select 5 specimens
      The number of ways 5 specimens can be selected from the 40 specimens is . To
      select 5 specimens, only 1 of which has the trait, you have to select 1 of the 3 specimens that have the trait and select 4 of the 37 specimens that do not have the trait. The number of such selections is the product . So the probability that only 1 of the 5 specimens selected will have the trait is represented by . The correct answer is Choice E.

      Directions: Answer the questions based on the following passage.
      Coffee has long been the subject of research due to its popularity as an early-morning pick-me-up and its distinct taste and aroma. Although it has often been laden with a reputation for being potentially unhealthy, many studies have shown that the opposite is true; in fact, coffee has been tied to a wide range of benefits. The acid in coffee can contribute to heartburn, and the caffeine can raise blood pressure, but when consumed in moderation (a few regular cups a day), these disadvantages are minimized.
      Although coffee was once linked to cancer, that association has long been dispelled. Instead, coffee may contribute to the prevention of certain types of cancers due to its high volume of antioxidants.
      Minerals found in coffee, like magnesium and chromium, help the body control blood sugar by influencing insulin, and this may contribute to preventing diabetes. Similarly, although researchers aren’t sure why, coffee drinkers seem to have a better chance than do non–coffee drinkers of fighting off Parkinson’s disease and dementia as they age. In the short term, coffee is low in calories, stimulates alertness and concentration, and, for some people, lengthens their attention spans.
      Q.The author would most likely disagree with which of the following statements?

      I. Stimulants are inherently bad for the human body.
      II. It is possible for coffee to be a part of a healthy diet.
      III. Decaffeinated coffee does not have the same benefits as caffeinated coffee.

       
      • a)
        II Only
      • b)
        I Only
      • c)
        III Only
      • d)
        I and II Only
      • e)
        II and III Only
      Correct answer is option 'B'. Can you explain this answer?

      Meera Rana answered
      While the passage does recognize one downside of caffeine as a stimulant (high blood pressure), it also points out a handful of benefits (higher levels of alertness and concentration as well as longer attention spans). Therefore, the author is likely to disagree with choice (A). It is likely that the author would agree with choice (B), since coffee has been “tied to a wide range of benefits.” Although there is no evidence to say that the author would agree with choice (C), because the author doesn’t address research on different types of coffee, you can’t infer anything about whether or not the author would disagree with that statement.

      Directions: Choose the letter that best answers the question.
      Q. Charles is able to husk 216 ears of corn in 2 ½ hours. What was his rate per minute?
      • a)
        1.44
      • b)
        5.4
      • c)
        14.4
      • d)
        54
      • e)
        144
      Correct answer is option 'A'. Can you explain this answer?

      Alok Saxena answered
      Given:
      - Charles husks 216 ears of corn in 2 ½ hours.

      To find:
      - Charles' rate per minute.

      Solution:
      To find Charles' rate per minute, we need to convert the time taken to husk the corn into minutes.

      Step 1: Convert 2 ½ hours into minutes.
      - There are 60 minutes in 1 hour.
      - So, 2 ½ hours = 2 * 60 + 30 = 120 + 30 = 150 minutes.

      Step 2: Calculate the rate per minute.
      - Rate = Amount / Time
      - Rate = 216 ears / 150 minutes
      - Rate = 1.44 ears/minute

      Therefore, Charles' rate per minute is 1.44.

      Hence, the correct answer is option A) 1.44.

      The Russians sought formal Western recognition of their expanded borders and acceptance of, or at least __________ in, their emerging sphere of influence in Eastern Europe.
      • a)
        desiccation
      • b)
        acquiescence
      • c)
        dissent
      • d)
        insubordination
      • e)
        surfeit
      Correct answer is option 'B'. Can you explain this answer?

      Alok Saxena answered
      Explanation:

      The correct answer is option B: acquiescence.

      Acquiescence:
      Acquiescence means accepting or agreeing to something without protest or resistance. In the given context, the Russians sought formal Western recognition of their expanded borders and acceptance of, or at least acquiescence in, their emerging sphere of influence in Eastern Europe.

      Reasoning:
      To understand why option B is the correct answer, let's analyze the other options:

      - Desiccation: Desiccation means the process of drying out. It is unrelated to the context and does not fit in the sentence.

      - Dissent: Dissent means to have a difference of opinion or to disagree. It is not the appropriate word in this context as the Russians are seeking recognition and acceptance, not disagreement.

      - Insubordination: Insubordination means refusing to obey orders or authority. It is not relevant in this context as the Russians are seeking recognition and acceptance, not defiance.

      - Surfeit: Surfeit means an excessive amount or overabundance. It does not make sense in this context as the sentence is referring to acceptance or recognition, not an excessive amount.

      Therefore, option B, acquiescence, is the correct answer as it means accepting or agreeing without protest, which aligns with the Russians seeking formal Western recognition of their expanded borders and acceptance in their emerging sphere of influence in Eastern Europe.

      Direction: Read the following passage carefully and answer the questions given below it. 
      An idea that has gained renewed currency in recent years is that the proliferation of information technology has a deleterious effect on interpersonal relationships. Neil Postman, in his book Amusing Ourselves to Death, asserted that television, as a medium, was incapable of fostering intelligent, meaningful discourse around a given subject. Postman wrote his book in the 1980s, when cable television was first becoming widely available. In it, he discusses the way that putatively “serious” news programs are inherently trite. Terse reports on serious, weighty issues such as the war in Iraq are juxtaposed with trivial information, such as celebrity gossip. Often the two are linked, one after another, by the phrase “and now. . .” This leaves the viewer unable to emotionally respond to something traumatic, as he is bombarded with disparate pieces of information in rapid succession.
      Instant access to a glut of information impoverishes genuine experiences, as it deprives the information of a meaningful context for interpretation.
      Q. Based on his views as they’re expressed in the passage, Postman would likely agree with which of the following statements?
      • a)
        Information technologies such as smartphones are unlikely to provide meaningful contexts in which to interpret information.
      • b)
        Television programs inherently lack intelligent, meaningful discourse.
      • c)
        The medium in which factual content is delivered can be an important factor in how that content is interpreted.
      Correct answer is option 'A,C'. Can you explain this answer?

      Meera Rana answered
      This question is asking you to infer Postman’s position from what the brief selection tells you about his views. Choice (A) is correct because the author noted that Postman thought “instant access to a glut of information” reduced meaningful context for that information. Thus, he would likely apply this reasoning to newer information technology that has the same effect. Choice (B) is subtly wrong. Postman, you’re told, thought television incapable of fostering intelligent discourse among its viewers; you cannot conclude that he therefore thinks that no intelligent discourse appears in any television program. In fact, part of Postman’s problem with the medium is that serious topics are juxtaposed with frivolous items. Postman would definitely agree with the statement in choice (C). Given his views on television, as a medium, and those in the final sentence of the passage, you can infer that he believes that media influence how information is interpreted.

      Which of the following integers satisfies this condition: It is a multiple of 8 and it is the cube of an even number? Indicate all such integers. [Multiple Seelct Answers]
      • a)
        72
      • b)
        64
      • c)
        216
      • d)
        1000
      • e)
        128
      Correct answer is option 'B,C,D'. Can you explain this answer?

      Meera Rana answered
      (A) 72 and (E) 128 are multiples of 8 but are not cubes of any integer. The following satisfy the given condition
      (B) 64 is a multiple of 8 (8 x 8) and the cube of 4 (4 x 4 x 4)
      (C) 216 is a multiple of 8 (8 x 27) and the cube of 6 (6 x 6 x 6)
      (D) 1000 is a multiple of 8 (8 x 125) and the cube of 10 (10 x 10 x 10)

      Directions: Answer the questions based on following reading passage.
      Observations of the Arctic reveal that the Arctic Ocean is covered by less ice each summer than the previous summer. If this warming trend continues, within 50 years the Arctic Ocean will be ice free during the summer months. This occurrence would in itself have little or no effect on global sea levels, since the melting of ice floating in water does not affect the water level. However, serious consequences to sea levels would eventually result, because __________.
      Q. Which of the following most logically completes the passage?
      • a)
        large masses of floating sea ice would continue to form in the wintertime
      • b)
        significant changes in Arctic sea temperatures would be accompanied by changes in sea temperatures in more temperate parts of the world
      • c)
        such a warm Arctic Ocean would trigger the melting of massive landbased glaciers in the Arctic
      • d)
        an ice-free Arctic Ocean would support a very different ecosystem than it does presently
      • e)
        in the spring, melting sea ice would cause more icebergs to be created and to drift south into shipping routes
      Correct answer is option 'C'. Can you explain this answer?

      Meera Rana answered
      To logically complete the passage’s open-ended “because,” something is needed that will explain why the continuation of the warming trend would have serious consequences for sea levels. The passage explains that the melting of the Arctic Ocean ice will not affect sea levels because the contribution that the water contained in that ice makes to sea levels is the same whether the water is frozen or liquid. But Choice C points to a way in which increasing temperatures in the Arctic could add water to the ocean, namely by melting ice on the land. So Choice C logically completes the passage and is the correct answer.
      Given that the passage has already explained that melting sea ice does not affect sea levels, the formation of sea ice described in Choice A does not explain why there would be consequences for sea levels.
      Choices B, D, and E all describe possible consequences of increased temperatures in the Arctic, but none of these consequences suggests a mechanism by which sea levels would change. So none of these options provides a logical completion for the passage.

      A reading list for a humanities course consists of 10 books, of which 4 are biographies and the rest are novels. Each student is required to read a selection of 4 books from the list, including 2 or more biographies. How many selections of 4 books satisfy the requirements?
      • a)
        60
      • b)
        115
      • c)
        130
      • d)
        144
      • e)
        195
      Correct answer is option 'B'. Can you explain this answer?

      Kevin Nguyen answered
      To solve this problem, we can use the concept of combinations and consider the different possibilities for selecting the books.

      Let's break down the requirements:
      - There are 10 books in total, 4 of which are biographies and the rest are novels.
      - Each student is required to read a selection of 4 books.
      - The selection must include at least 2 biographies.

      To find the number of selections that satisfy these requirements, we can consider the possible combinations of biographies and novels.

      1. Selecting the biographies:
      Since we need to include at least 2 biographies, we can choose 2, 3, or all 4 biographies.

      - Choosing 2 biographies: This can be done in C(4, 2) = 6 ways.
      - Choosing 3 biographies: This can be done in C(4, 3) = 4 ways.
      - Choosing all 4 biographies: This can be done in C(4, 4) = 1 way.

      Therefore, there are 6 + 4 + 1 = 11 possible ways to select the biographies.

      2. Selecting the novels:
      After selecting the biographies, we need to choose the remaining novels. Since there are 6 novels in total (10 books - 4 biographies), we need to select 2 novels from these 6.

      This can be done in C(6, 2) = 15 ways.

      3. Combining the selections:
      Now, we can combine the selections of biographies and novels. For each selection of biographies, there are 15 possible selections of novels. Therefore, the total number of selections is 11 (selections of biographies) * 15 (selections of novels) = 165.

      However, we need to keep in mind that each student is required to read only 4 books. So, if we consider the order in which the books are selected, we would be overcounting the selections. To avoid this, we need to divide the total number of selections by the number of ways the 4 books can be arranged, which is 4!.

      Therefore, the final number of selections satisfying the requirements is 165 / 4! = 165 / 24 = 6.875.

      Since the number of selections must be a whole number, the nearest whole number to 6.875 is 7.

      Hence, the correct answer is option B) 115.

      Chapter doubts & questions for Practice Tests - GRE Mock Test Series 2025 2025 is part of GRE exam preparation. The chapters have been prepared according to the GRE exam syllabus. The Chapter doubts & questions, notes, tests & MCQs are made for GRE 2025 Exam. Find important definitions, questions, notes, meanings, examples, exercises, MCQs and online tests here.

      Chapter doubts & questions of Practice Tests - GRE Mock Test Series 2025 in English & Hindi are available as part of GRE exam. Download more important topics, notes, lectures and mock test series for GRE Exam by signing up for free.

      Top Courses GRE